4
$\begingroup$

We say a field $F$ has the property $*$ if the equation $x^2 + y^2=-1$ has no solution in $F$. For an example if $F$ is a subfield of real numbers then $F$ satisfies $*$. On the other hand if $ F $ is a finite field then by Chevalley-Warning theorem the equation $ x^2 + y^2 =-1 $ always has a solution. I want to know which fields that satisfy property $ * $. Does this type of field belong to a special class of field?

$\endgroup$
5
  • 1
    $\begingroup$ For what it's worth, these are precisely the fields over which the quaternion group $Q_8$ has a four dimensional irreducible representation. You might also want to look up the theory of Severi-Brauer varieties. $\endgroup$ Jul 10 at 6:51
  • $\begingroup$ "Additionally, it is assumed": you don't need this assumption, which is an immediate consequence of the definition. $\endgroup$
    – YCor
    Jul 10 at 7:02
  • $\begingroup$ Yes we don't need this assumption. I have edited. $\endgroup$
    – Sky
    Jul 10 at 7:14
  • 2
    $\begingroup$ There are many such fields with rather disparate properties, and I don’t think there is any kind of classification. FWIW, these are called the fields of Stufe 2. $\endgroup$ Jul 10 at 7:37
  • 1
    $\begingroup$ I mean, of Stufe more than $2$. $\endgroup$ Jul 10 at 7:46

1 Answer 1

9
$\begingroup$

In the notation of Lam's Quadratic forms over fields, the Stufe (a German word) or level (its English translation) $s(F)$ of a field is the minimal $n$ such that $-1$ is the sum of $n$ squares.

A theorem of Pfister says that $s(F)$ is always a power of $2$ (or $\infty$).

So you are taking of fields $F$ such that $s(F) > 2$ (or $s(F) \geq 4$, if you are willing to use Pfister's theorem).

$\endgroup$
2
  • 2
    $\begingroup$ Why would you be not willing to use Pfister's theorem? In this special case, it is an immediate consequence of the Brahmagupta–Fibonacci identity, hence it is has a short, completely elementary proof. $\endgroup$ Jul 10 at 8:26
  • $\begingroup$ What will be the stuff for padic field? $\endgroup$
    – Sky
    Jul 30 at 18:52

Your Answer

By clicking “Post Your Answer”, you agree to our terms of service and acknowledge that you have read and understand our privacy policy and code of conduct.

Not the answer you're looking for? Browse other questions tagged or ask your own question.